Skip to content

Commit

Permalink
Sections 4 and 5 inverted. Introduced \ before in.
Browse files Browse the repository at this point in the history
  • Loading branch information
nicola-pagani committed Nov 22, 2010
1 parent 8b66da2 commit fcde629
Showing 1 changed file with 136 additions and 136 deletions.
272 changes: 136 additions & 136 deletions doc/ProofOfGeneration.tex
Expand Up @@ -237,8 +237,8 @@ \section{How the program generates graphs}
\end{itemize}
So, for example, denote with $c_j$ and $c_{j+1}$ two adjacent columns
of the matrix~\ref{eq:big matrix}. They are said to be equal if
$c_{j,i} = c_{j+1,i}$ for any $i \not in \{j, j+1\}$; if they are not
equal, denote with $i_0$ the minimum index such that $i_0 \not in \{
$c_{j,i} = c_{j+1,i}$ for any $i \notin \{j, j+1\}$; if they are not
equal, denote with $i_0$ the minimum index such that $i_0 \notin \{
j+3, j+1+3\}$ and $c_{j,i_0} \neq c_{j+1,i_0}$. Then $c_j < c_{j+1}$
if and only if $c_{j,i_0} < c_{j+1,i_0}$. We do not define the
relation for non-adjacent columns.
Expand Down Expand Up @@ -394,140 +394,6 @@ \subsection{Range for $g_i$}
\end{align*}
\end{enumerate}

\subsection{Range for $n_i$}

When deciding $n_i$, we have the following situation:
\begin{itemize}
\item as before, $e^{\MAX} = G - G_K + K - 1 \geq K-1$, and the
maximum number of half edges still to be assigned and coming from
edges is $2e^{\MAX} - K = 2(G - G_K) + K - 2$;
\item the number of half edges still to be assigned and coming from
marked points is $N - N_i - n_i$;
\item we need $2p_1 - N^{(2)}_i - n^{(2)}_i$ half edges to
stabilize the first $p_1$ vertices;
\item if $g_i = 0$, we need $2(i+1) - N^{(2)}_i - n^{(2)}_i$ half
edges to stabilize the first $i+1$ vertices, and we cannot use
marked points.
\end{itemize}

The following conditions define then the ranges for the possible
choices for $n_i$:

\begin{enumerate}
\item if there is not a division before $i$ (that is, if $g_i =
g_{i-1}$), then we require $n_i \geq n_{i-1}$; otherwise, just $n_i
\geq 0$;
\item we cannot assign more than $N$ marked points, hence (where we
treat the case of $g_i = 0$ in a special way)
\begin{align*}
&N_i + n_i \leq N\\
&\qquad\Rightarrow n_i \leq N - N_i\\
&\qquad\Rightarrow (p_1 - i)n_i \leq N - N_i\text{ if moreover $g_i = 0$.}
\end{align*}
\item if $g_i = 0$, for the purpose of stabilizing the first $i+1$
curves we cannot use marked points anymore, therefore we have
\begin{align*}
&2 (i+1) - N^{(2)}_i - n^{(2)}_i \leq (2(G - G_K) + K - 2)\\
&\qquad\Rightarrow n^{(2)}_i = \min(2, n_i) \geq - (2(G - G_K) + K - 2) + (2(i+1) - N^{(2)}_i)\\
&\qquad\Rightarrow
\begin{cases}
\text{impossible} & \text{if $\mathrm{RHS} > 2$}\\
n_i \geq \mathrm{RHS} & \text{otherwise.}
\end{cases}
\end{align*}
\end{enumerate}

\subsection{Range for $l_i$}

After deciding $l_i$, this is the situation:
\begin{itemize}
\item $e^{\MAX} = G - G_K - L_i - l_i + K - 1 \geq K-1$, and the
maximum number of half edges coming from edges to assign is
$2e^{\MAX} - K = 2(G - G_K - L_i - l_i) + K - 2$;
\end{itemize}

The conditions on $l_i$ are then the following:
\begin{enumerate}
\item if there is not a division before $i$, then we require $l_i \geq
l_{i-1}$; otherwise, just $l_i \geq 0$;
\item we need at least $K-1$ non-loop edges, hence
\begin{align*}
&e^{\MAX} \geq K-1\\
&\qquad\Rightarrow G - G_K - L_i - l_i + K-1 \geq K-1\\
&\qquad\Rightarrow l_i \leq G - G_K - L_i\,\text{;}
\end{align*}
\item let $z$ be the index of the genus $0$ vertex with the least
number of stabilizing half edges such that $z < i$; it already has
$n_z + 2l_z$ half edges, but we cannot use loops anymore to
stabilize it; hence,
\begin{align*}
&\max(0, 2-n_z-2l_z) \leq G - G_K - L_i - l_i + K - 1\\
&\qquad\Rightarrow l_i \leq G - G_k - L_i + K - 3 + n_z + 2l_z
\end{align*}
\item assume $g_i = 0$; if $l_i > 0$, we are adding to the $i$-th
vertex $2-n^{(2)}_i$ stabilizing half edges, and to stabilize the
$p_1$ genus $0$ vertices, we need to have
\begin{align*}
&2 p_1 - N^{(2)} - (2-n^{(2)}_i) \leq 2e^{\MAX} - K\\
&\qquad\Rightarrow 2p_1 - N^{(2)} - (2-n^{(2)}_i) \max(0, 2-m_i) \leq 2(G - G_K - L_i - l_i + K - 1) - K\\
&\qquad\Rightarrow 2l_i \leq 2(G - G_K - L_i) + K + N^{(2)} - n^{(2)}_i - 2p_i\,\text{.}
\end{align*}
\item assume $g_i = 0$; after deciding $l_i$, we still have $e^{\MAX}$
edges to place, and each of them can contribute with one half edge
to the stabilization of the $i$-th vertex; moreover, one of that
half edges is already counted toward the stabilization; hence
\begin{align*}
&n_i + 2l_i + (e^{\MAX} - 1) \geq 2\\
&\qquad\Rightarrow n_i + 2l_i + G - G_K - L_i - l_i + K - 1 - 1 \geq 2\\
&\qquad\Rightarrow l_i \geq 4 - n_i - G + G_K + L_i - K\,\text{.}
\end{align*}
\end{enumerate}

\subsection{Range for $a_{i,j}$}

After deciding $a_{i,j}$, this is the situation:
\begin{itemize}
\item in this section, the word ``stabilized'' goes back to its
original meaning, i.e., it means ``with at least $3$ half edges'';
this is necessary since here we may have already placed some
non-loop edges, hence we cannot track easily which vertices already
are connected to the rest of the graph and which are not;
\item $e^{\MAX} = G - G_K - L_K - A_{i,j} + K - 1$;
\item we have already placed edges between $c$ couples of different
vertices;
\end{itemize}

Here are the constraints that $a_{i,j}$ must satisfy:
\begin{enumerate}
\item if there is not a division before $i$, then we require $a_{i,j} \geq
a_{i-1,j}$; otherwise, just $a_{i,j} \geq 0$;
\item if there is not a division before $j$, then we require $a_{i,j}
\geq a_{i,j-1}$;
\item we need at least $K-2-c$ (if positive) edges to connect the
graph, because if $a_{i,j} > 0$, $c$ will increase by $1$ (this
estimate could be very poor, but enforcing the connectedness
condition in its entirety before completing the graph is too slow),
hence:
\begin{align*}
&e^{\MAX} - a_{i,j} \geq \max(0, K-2-c)\\
&\qquad\Rightarrow a_{i,j} \leq G - G_K - L_K - A_{i,j} +K - 1 - \max(0, K-2-c)\,\text{;}
\end{align*}
\item $a_{i,j}$ contributes with at most $\max(0, 3-h_i) + \max(0,
3-h_j)$ stabilizing half edges; hence, to stabilize the $p_1$ genus
$0$ vertices, we need
\begin{align*}
&3p_1 - \sum_{g_{i^\prime} = 0} \min(3, n_i + 2l_i) - (\max(0, 3-h_i) + \max(0, 3-h_j)) \leq 2 (e^{\MAX} - a_{i,j})\\
&\qquad\Rightarrow 3p_1 - \sum_{g_{i^\prime} = 0} \min(3, n_i + 2l_i) - (\max(0, 3-h_i) + \max(0, 3-h_j)) \leq \\
&\qquad\qquad\qquad \leq 2 (G - G_K - L_K - A_{i,j} + K - 1 - a_{i,j})\\
&\qquad\Rightarrow 2a_{i,j} \leq 2 (G - G_K - L_K - A_{i,j} + K - 1) - 3p_1 +\\
&\qquad\qquad\qquad +\sum_{g_{i^\prime} = 0} \min(3, n_i + 2l_i) + \max(0, 3-h_i) + \max(0, 3-h_j)\,\text{.}
\end{align*}
\item if $j = K-1$ (that is, if this is the last chance to add half
edges to the $i$-th vertex), then we add enough edges from $i$ to
$K-1$ in order to stabilize it; moreover, if up to now we did not
place any non-loop edge on it, we impose $a_{i,K-1} > 0$.
\end{enumerate}



\section{The program generates all graphs}\label{sec:proof}
Expand Down Expand Up @@ -890,6 +756,140 @@ \section{The program generates all graphs}\label{sec:proof}
is finite.
\end{proof}

\subsection{Range for $n_i$}

When deciding $n_i$, we have the following situation:
\begin{itemize}
\item as before, $e^{\MAX} = G - G_K + K - 1 \geq K-1$, and the
maximum number of half edges still to be assigned and coming from
edges is $2e^{\MAX} - K = 2(G - G_K) + K - 2$;
\item the number of half edges still to be assigned and coming from
marked points is $N - N_i - n_i$;
\item we need $2p_1 - N^{(2)}_i - n^{(2)}_i$ half edges to
stabilize the first $p_1$ vertices;
\item if $g_i = 0$, we need $2(i+1) - N^{(2)}_i - n^{(2)}_i$ half
edges to stabilize the first $i+1$ vertices, and we cannot use
marked points.
\end{itemize}

The following conditions define then the ranges for the possible
choices for $n_i$:

\begin{enumerate}
\item if there is not a division before $i$ (that is, if $g_i =
g_{i-1}$), then we require $n_i \geq n_{i-1}$; otherwise, just $n_i
\geq 0$;
\item we cannot assign more than $N$ marked points, hence (where we
treat the case of $g_i = 0$ in a special way)
\begin{align*}
&N_i + n_i \leq N\\
&\qquad\Rightarrow n_i \leq N - N_i\\
&\qquad\Rightarrow (p_1 - i)n_i \leq N - N_i\text{ if moreover $g_i = 0$.}
\end{align*}
\item if $g_i = 0$, for the purpose of stabilizing the first $i+1$
curves we cannot use marked points anymore, therefore we have
\begin{align*}
&2 (i+1) - N^{(2)}_i - n^{(2)}_i \leq (2(G - G_K) + K - 2)\\
&\qquad\Rightarrow n^{(2)}_i = \min(2, n_i) \geq - (2(G - G_K) + K - 2) + (2(i+1) - N^{(2)}_i)\\
&\qquad\Rightarrow
\begin{cases}
\text{impossible} & \text{if $\mathrm{RHS} > 2$}\\
n_i \geq \mathrm{RHS} & \text{otherwise.}
\end{cases}
\end{align*}
\end{enumerate}

\subsection{Range for $l_i$}

After deciding $l_i$, this is the situation:
\begin{itemize}
\item $e^{\MAX} = G - G_K - L_i - l_i + K - 1 \geq K-1$, and the
maximum number of half edges coming from edges to assign is
$2e^{\MAX} - K = 2(G - G_K - L_i - l_i) + K - 2$;
\end{itemize}

The conditions on $l_i$ are then the following:
\begin{enumerate}
\item if there is not a division before $i$, then we require $l_i \geq
l_{i-1}$; otherwise, just $l_i \geq 0$;
\item we need at least $K-1$ non-loop edges, hence
\begin{align*}
&e^{\MAX} \geq K-1\\
&\qquad\Rightarrow G - G_K - L_i - l_i + K-1 \geq K-1\\
&\qquad\Rightarrow l_i \leq G - G_K - L_i\,\text{;}
\end{align*}
\item let $z$ be the index of the genus $0$ vertex with the least
number of stabilizing half edges such that $z < i$; it already has
$n_z + 2l_z$ half edges, but we cannot use loops anymore to
stabilize it; hence,
\begin{align*}
&\max(0, 2-n_z-2l_z) \leq G - G_K - L_i - l_i + K - 1\\
&\qquad\Rightarrow l_i \leq G - G_k - L_i + K - 3 + n_z + 2l_z
\end{align*}
\item assume $g_i = 0$; if $l_i > 0$, we are adding to the $i$-th
vertex $2-n^{(2)}_i$ stabilizing half edges, and to stabilize the
$p_1$ genus $0$ vertices, we need to have
\begin{align*}
&2 p_1 - N^{(2)} - (2-n^{(2)}_i) \leq 2e^{\MAX} - K\\
&\qquad\Rightarrow 2p_1 - N^{(2)} - (2-n^{(2)}_i) \max(0, 2-m_i) \leq 2(G - G_K - L_i - l_i + K - 1) - K\\
&\qquad\Rightarrow 2l_i \leq 2(G - G_K - L_i) + K + N^{(2)} - n^{(2)}_i - 2p_i\,\text{.}
\end{align*}
\item assume $g_i = 0$; after deciding $l_i$, we still have $e^{\MAX}$
edges to place, and each of them can contribute with one half edge
to the stabilization of the $i$-th vertex; moreover, one of that
half edges is already counted toward the stabilization; hence
\begin{align*}
&n_i + 2l_i + (e^{\MAX} - 1) \geq 2\\
&\qquad\Rightarrow n_i + 2l_i + G - G_K - L_i - l_i + K - 1 - 1 \geq 2\\
&\qquad\Rightarrow l_i \geq 4 - n_i - G + G_K + L_i - K\,\text{.}
\end{align*}
\end{enumerate}

\subsection{Range for $a_{i,j}$}

After deciding $a_{i,j}$, this is the situation:
\begin{itemize}
\item in this section, the word ``stabilized'' goes back to its
original meaning, i.e., it means ``with at least $3$ half edges'';
this is necessary since here we may have already placed some
non-loop edges, hence we cannot track easily which vertices already
are connected to the rest of the graph and which are not;
\item $e^{\MAX} = G - G_K - L_K - A_{i,j} + K - 1$;
\item we have already placed edges between $c$ couples of different
vertices;
\end{itemize}

Here are the constraints that $a_{i,j}$ must satisfy:
\begin{enumerate}
\item if there is not a division before $i$, then we require $a_{i,j} \geq
a_{i-1,j}$; otherwise, just $a_{i,j} \geq 0$;
\item if there is not a division before $j$, then we require $a_{i,j}
\geq a_{i,j-1}$;
\item we need at least $K-2-c$ (if positive) edges to connect the
graph, because if $a_{i,j} > 0$, $c$ will increase by $1$ (this
estimate could be very poor, but enforcing the connectedness
condition in its entirety before completing the graph is too slow),
hence:
\begin{align*}
&e^{\MAX} - a_{i,j} \geq \max(0, K-2-c)\\
&\qquad\Rightarrow a_{i,j} \leq G - G_K - L_K - A_{i,j} +K - 1 - \max(0, K-2-c)\,\text{;}
\end{align*}
\item $a_{i,j}$ contributes with at most $\max(0, 3-h_i) + \max(0,
3-h_j)$ stabilizing half edges; hence, to stabilize the $p_1$ genus
$0$ vertices, we need
\begin{align*}
&3p_1 - \sum_{g_{i^\prime} = 0} \min(3, n_i + 2l_i) - (\max(0, 3-h_i) + \max(0, 3-h_j)) \leq 2 (e^{\MAX} - a_{i,j})\\
&\qquad\Rightarrow 3p_1 - \sum_{g_{i^\prime} = 0} \min(3, n_i + 2l_i) - (\max(0, 3-h_i) + \max(0, 3-h_j)) \leq \\
&\qquad\qquad\qquad \leq 2 (G - G_K - L_K - A_{i,j} + K - 1 - a_{i,j})\\
&\qquad\Rightarrow 2a_{i,j} \leq 2 (G - G_K - L_K - A_{i,j} + K - 1) - 3p_1 +\\
&\qquad\qquad\qquad +\sum_{g_{i^\prime} = 0} \min(3, n_i + 2l_i) + \max(0, 3-h_i) + \max(0, 3-h_j)\,\text{.}
\end{align*}
\item if $j = K-1$ (that is, if this is the last chance to add half
edges to the $i$-th vertex), then we add enough edges from $i$ to
$K-1$ in order to stabilize it; moreover, if up to now we did not
place any non-loop edge on it, we impose $a_{i,K-1} > 0$.
\end{enumerate}

% TODO: write a new section with some data obtained by the program,
% and consideration on running time (e.g.: running time / found
% graphs)
Expand Down

0 comments on commit fcde629

Please sign in to comment.